Do working physicists consider Newtonian mechanics to be “falsified”?












21












$begingroup$


In the comments for the question Falsification in Math vs Science, a dispute around the question of "Have Newtonian Mechanics been falsified?"



That's a bit of a vague question, so attempting to narrow it a bit:




  1. Are any of Newton's three laws considered to be 'falsified theories' by any 'working physicists'? If so, what evidence do they have that they believe falsifies those three theories?


  2. If the three laws are still unfalsified, are there any other concepts that form a part of "Newtonian Mechanics" that we consider to be falsified?











share|cite|improve this question









New contributor




Pod is a new contributor to this site. Take care in asking for clarification, commenting, and answering.
Check out our Code of Conduct.







$endgroup$












  • $begingroup$
    Related: physics.stackexchange.com/questions/52165/…
    $endgroup$
    – user3067860
    Apr 12 at 17:37






  • 13




    $begingroup$
    "All models are wrong, but some are useful"
    $endgroup$
    – Eric Duminil
    Apr 12 at 20:34






  • 1




    $begingroup$
    @EricDuminil The quotes by statisticians better be reserved for statistics. (-:
    $endgroup$
    – Dvij Mankad
    Apr 13 at 23:23










  • $begingroup$
    @DvijMankad Science relies on statistics.
    $endgroup$
    – Schwern
    Apr 15 at 3:41






  • 1




    $begingroup$
    @Schwern No, of course. It was just a bad joke.
    $endgroup$
    – Dvij Mankad
    2 days ago
















21












$begingroup$


In the comments for the question Falsification in Math vs Science, a dispute around the question of "Have Newtonian Mechanics been falsified?"



That's a bit of a vague question, so attempting to narrow it a bit:




  1. Are any of Newton's three laws considered to be 'falsified theories' by any 'working physicists'? If so, what evidence do they have that they believe falsifies those three theories?


  2. If the three laws are still unfalsified, are there any other concepts that form a part of "Newtonian Mechanics" that we consider to be falsified?











share|cite|improve this question









New contributor




Pod is a new contributor to this site. Take care in asking for clarification, commenting, and answering.
Check out our Code of Conduct.







$endgroup$












  • $begingroup$
    Related: physics.stackexchange.com/questions/52165/…
    $endgroup$
    – user3067860
    Apr 12 at 17:37






  • 13




    $begingroup$
    "All models are wrong, but some are useful"
    $endgroup$
    – Eric Duminil
    Apr 12 at 20:34






  • 1




    $begingroup$
    @EricDuminil The quotes by statisticians better be reserved for statistics. (-:
    $endgroup$
    – Dvij Mankad
    Apr 13 at 23:23










  • $begingroup$
    @DvijMankad Science relies on statistics.
    $endgroup$
    – Schwern
    Apr 15 at 3:41






  • 1




    $begingroup$
    @Schwern No, of course. It was just a bad joke.
    $endgroup$
    – Dvij Mankad
    2 days ago














21












21








21


7



$begingroup$


In the comments for the question Falsification in Math vs Science, a dispute around the question of "Have Newtonian Mechanics been falsified?"



That's a bit of a vague question, so attempting to narrow it a bit:




  1. Are any of Newton's three laws considered to be 'falsified theories' by any 'working physicists'? If so, what evidence do they have that they believe falsifies those three theories?


  2. If the three laws are still unfalsified, are there any other concepts that form a part of "Newtonian Mechanics" that we consider to be falsified?











share|cite|improve this question









New contributor




Pod is a new contributor to this site. Take care in asking for clarification, commenting, and answering.
Check out our Code of Conduct.







$endgroup$




In the comments for the question Falsification in Math vs Science, a dispute around the question of "Have Newtonian Mechanics been falsified?"



That's a bit of a vague question, so attempting to narrow it a bit:




  1. Are any of Newton's three laws considered to be 'falsified theories' by any 'working physicists'? If so, what evidence do they have that they believe falsifies those three theories?


  2. If the three laws are still unfalsified, are there any other concepts that form a part of "Newtonian Mechanics" that we consider to be falsified?








newtonian-mechanics models






share|cite|improve this question









New contributor




Pod is a new contributor to this site. Take care in asking for clarification, commenting, and answering.
Check out our Code of Conduct.











share|cite|improve this question









New contributor




Pod is a new contributor to this site. Take care in asking for clarification, commenting, and answering.
Check out our Code of Conduct.









share|cite|improve this question




share|cite|improve this question








edited Apr 13 at 23:07









MarianD

329129




329129






New contributor




Pod is a new contributor to this site. Take care in asking for clarification, commenting, and answering.
Check out our Code of Conduct.









asked Apr 12 at 8:07









PodPod

21525




21525




New contributor




Pod is a new contributor to this site. Take care in asking for clarification, commenting, and answering.
Check out our Code of Conduct.





New contributor





Pod is a new contributor to this site. Take care in asking for clarification, commenting, and answering.
Check out our Code of Conduct.






Pod is a new contributor to this site. Take care in asking for clarification, commenting, and answering.
Check out our Code of Conduct.












  • $begingroup$
    Related: physics.stackexchange.com/questions/52165/…
    $endgroup$
    – user3067860
    Apr 12 at 17:37






  • 13




    $begingroup$
    "All models are wrong, but some are useful"
    $endgroup$
    – Eric Duminil
    Apr 12 at 20:34






  • 1




    $begingroup$
    @EricDuminil The quotes by statisticians better be reserved for statistics. (-:
    $endgroup$
    – Dvij Mankad
    Apr 13 at 23:23










  • $begingroup$
    @DvijMankad Science relies on statistics.
    $endgroup$
    – Schwern
    Apr 15 at 3:41






  • 1




    $begingroup$
    @Schwern No, of course. It was just a bad joke.
    $endgroup$
    – Dvij Mankad
    2 days ago


















  • $begingroup$
    Related: physics.stackexchange.com/questions/52165/…
    $endgroup$
    – user3067860
    Apr 12 at 17:37






  • 13




    $begingroup$
    "All models are wrong, but some are useful"
    $endgroup$
    – Eric Duminil
    Apr 12 at 20:34






  • 1




    $begingroup$
    @EricDuminil The quotes by statisticians better be reserved for statistics. (-:
    $endgroup$
    – Dvij Mankad
    Apr 13 at 23:23










  • $begingroup$
    @DvijMankad Science relies on statistics.
    $endgroup$
    – Schwern
    Apr 15 at 3:41






  • 1




    $begingroup$
    @Schwern No, of course. It was just a bad joke.
    $endgroup$
    – Dvij Mankad
    2 days ago
















$begingroup$
Related: physics.stackexchange.com/questions/52165/…
$endgroup$
– user3067860
Apr 12 at 17:37




$begingroup$
Related: physics.stackexchange.com/questions/52165/…
$endgroup$
– user3067860
Apr 12 at 17:37




13




13




$begingroup$
"All models are wrong, but some are useful"
$endgroup$
– Eric Duminil
Apr 12 at 20:34




$begingroup$
"All models are wrong, but some are useful"
$endgroup$
– Eric Duminil
Apr 12 at 20:34




1




1




$begingroup$
@EricDuminil The quotes by statisticians better be reserved for statistics. (-:
$endgroup$
– Dvij Mankad
Apr 13 at 23:23




$begingroup$
@EricDuminil The quotes by statisticians better be reserved for statistics. (-:
$endgroup$
– Dvij Mankad
Apr 13 at 23:23












$begingroup$
@DvijMankad Science relies on statistics.
$endgroup$
– Schwern
Apr 15 at 3:41




$begingroup$
@DvijMankad Science relies on statistics.
$endgroup$
– Schwern
Apr 15 at 3:41




1




1




$begingroup$
@Schwern No, of course. It was just a bad joke.
$endgroup$
– Dvij Mankad
2 days ago




$begingroup$
@Schwern No, of course. It was just a bad joke.
$endgroup$
– Dvij Mankad
2 days ago










6 Answers
6






active

oldest

votes


















45












$begingroup$

"Falsified" is more philosophical than scientific distinction.
Newton laws have been falsified somehow, but we still use them, since usually they are a good approximation, and are easier to use than relativity or quantum mechanics.



The "action at distance" of Newton potentials has been falsified (finite speed of light...) but again, we use it every day.



So, in practical terms, no, Newton laws are still not falsified, in the sense that are not totally discredited in the scientific community. Classical mechanics is still in the curriculum of all universities, in a form more or less identical that 200 years ago (Before Relativity, quantum mechanics, field theory).



Most concept in physics fit more in the category of "methods" rather than "paradigms", so can be used over and over again. And all current methods and laws fails and give "false" results, when used outside their range of applicability.



The typical example of "falsified" theory is the Ptolemaic system of Sun & planets rotating around the Earth. However, philosopher usually omits the facts that:




  • Ptolemaic system was experimentally pretty good at calculating planet motions

  • Most mathematical and experimental methods of the new Heliocentric paradigm are the same of the old Ptolemaic


So the falsification was more on the point of view, rather than in the methods.






share|cite|improve this answer











$endgroup$









  • 3




    $begingroup$
    Well, we can measure that the Earth is rotating, against Ptolemaic system. But yes, we can still write ( with a lot of patience) all physics from our rotating system, with a lot of "fictitious" forces.
    $endgroup$
    – patta
    Apr 12 at 9:30








  • 20




    $begingroup$
    Note that both general mechanics and quantum mechanics are just as false as Newtonian mechanics, in a certain sense. QM fails to predict gravitational lensing, and GR fails to predict interference patterns in the double slit experiment. Both fail to explain how black holes preserve information. (one says they don't, the other says they don't exist)
    $endgroup$
    – John Dvorak
    Apr 12 at 10:04








  • 17




    $begingroup$
    @piet.t That's a common misconception. Ptolemy's model actually makes physically different predictions. Note that Venus is always between the Earth and the Sun in the Ptolemaic model, but not in reality. Galileo's observation of the phases of Venus, showing Venus fully illuminated by the sun, falsified the Ptolemaic model.
    $endgroup$
    – Denziloe
    Apr 12 at 11:08






  • 8




    $begingroup$
    99 c = 1 $ $pm$ 2%
    $endgroup$
    – patta
    Apr 12 at 18:41








  • 4




    $begingroup$
    @piet.t, the Ptolemaic system predicts that the fixed stars remain in the same relative locations at all times. Heliocentric models predict that they'll show annual variations in their positions relative to each other. High-precision observations of stellar positions show both parallax and aberration of light, which rather falsifies the Ptolemaic system (and almost all other geocentric systems).
    $endgroup$
    – Mark
    Apr 12 at 20:47





















29












$begingroup$

Newtonian Physics is accurate in the specific domain it was designed for



Physics is not about identifying the "truth" of the world around us. It's about creating mathematical models that allow us to accurately predict the behavior of the world.



Nobody is trying to create a perfect model, because the complexity of such a model would be infinite. Instead, we look for the boundaries of a model's accuracy - under what conditions it produces reasonable results, and the precision of the results it produces under those conditions.



You can see this more clearly with other physics models, such as the Ideal Gas Law. The Ideal Gas Law models a hugely complex system of particle collisions as a simple formula of ratios. It breaks down relatively quickly at high or low values of any of its quantities, but because we understand when and how the law breaks down, it is still useful.



At extremely large quantities (large speeds, large masses, high energies), the Newtonian model starts to break down, and we need to use a Relativistic model in order to get accurate results. But that doesn't mean that the Newtonian model is false, it just means that it is inapplicable for those conditions.



Obviously, Newton wasn't aware of the limitations to his laws when he described them. He was trying to create a universally applicable set of relations. In that sense you could argue that he failed. But I would consider the modern understanding a refinement of his laws, rather than a falsification.






share|cite|improve this answer











$endgroup$









  • 1




    $begingroup$
    Avoids answering the question...
    $endgroup$
    – Rob Jeffries
    Apr 13 at 16:53






  • 6




    $begingroup$
    This is the modern understanding, but I don't think it is true to say that Newton's laws were designed (by Newton, at least) for slow speeds and mild gravitational curvatures.
    $endgroup$
    – Rococo
    Apr 13 at 19:36






  • 5




    $begingroup$
    This answer would probably make more sense if "designed for" was replaced with "conceptualised within".
    $endgroup$
    – Ian Kemp
    Apr 13 at 22:37






  • 2




    $begingroup$
    @IanKemp I would've awarded a bounty on this comment if it were a feature! I mean, of course, Newtonian mechanics was designed for everything. It was just conceptualized within a certain regime of experiments. And it is not in the spirit of science to actually design a theory for the regime in which the experimental results are already known. A scientific theory has to make predictions and that means that it necessarily has to go beyond the domain from which it takes empirical inspiration.
    $endgroup$
    – Dvij Mankad
    Apr 13 at 23:33










  • $begingroup$
    When Newton was studying Kepler's work, he probably wasn't wondering if he could apply it to the $1s$ orbital of the hydrogen atom.
    $endgroup$
    – Cinaed Simson
    2 days ago





















9












$begingroup$

One of the problems of Newton's law of universal gravitation, $$F_text{Grav} = G frac{m_1m_2}{r^2},$$ is that it does not correctly describe the precession of Mercury's orbit. Mercury behaves slightly different than predicted by Newton's law and general relativity does a better job.



See also the corresponding Wikipedia article.






share|cite|improve this answer











$endgroup$









  • 2




    $begingroup$
    When you say "Newton's Laws", which do you mean exactly? There are laws for inertial motion, action/reaction, force as dp/dt, and gravity. I believe only the last one could be seen as needing modification by General Relativity.
    $endgroup$
    – Jens
    Apr 13 at 10:55










  • $begingroup$
    Thanks, clarified.
    $endgroup$
    – Jasper
    Apr 13 at 16:49










  • $begingroup$
    Indeed Mercury's elliptical orbit slowly rotates by a tiny amount extra than what Newton's gravity predicts. However people generally underemphasise other precession effects: 5000 "/c [seconds of arc per century] from precession of the equinoxes, and 530 "/c from other planets, compared with the observed 43 "/c extra that general relativity (and other gravity theories) explains. So Newtonian gravity is correct to within less than 1% error, in explaining the precession of Mercury's orbit.
    $endgroup$
    – Colin MacLaurin
    yesterday



















2












$begingroup$

Relativity is an extension of Newtonian physics, not either a replacement or correction. As such, relativity does not "falsify" Newtonian physics. For velocities far smaller than the speed of light (approaching zero), relativity simplifies back to the Newtonian model. For everyday use, and for everyday engineering problems, Newtonian physics is more than accurate enough. It's only when you get into more "interesting" situations that Newtonian physics fails to provide adequate solutions. The orbit of Mercury is a famous one. It's only because of Mercury's proximity to the Sun that its orbit defies accurate modeling in purely Newtonian terms. Similarly, without an understanding of relativity and relativistic effects on orbiting spacecraft, the GPS system could not work (the onboard timekeeping of the GPS satellites must be extremely precise and the very small relativisitic effects on their clocks must be accounted for). These are not everyday situations, and the relativistic effects are small, but the position of Mercury can be very precisely measured and GPS signals are timed with very high precision (light/radio travels about a foot or about 30cm in a nanosecond).






share|cite|improve this answer









$endgroup$













  • $begingroup$
    This answer seems to flip between two contradictory viewpoints. You say relativity is an "extension" of Newtonian physics, not a replacement or correction; but then proceed to talk about Newtonian physics "failing to provide adequate solutions" unless relativity is accounted for. Wouldn't that make it a "correction" to Newtonian physics in most senses of the term? I wouldn't consider new information that makes all prior information slightly inaccurate an "extension"; I would specifically call it a correction; because it corrects the errors in Newtonian physics.
    $endgroup$
    – JMac
    2 days ago










  • $begingroup$
    @JMac I stand by my core point that relativity is an extension not a correction because of the way the relativistic terms cancel out (leaving the simplified Newtonian forms) as velocity approaches zero (or the comparable terms which relate to curvature of spacetime due to mass).
    $endgroup$
    – Anthony X
    2 days ago





















1












$begingroup$

First of all no scientific theory can possibly be falsified. Popper was wrong. See the Quine-Duhem thesis which says that instead of rejecting the theory when a seemingly falsifying experiment occurs, one can always instead reject some underlying "auxiliary hypothesis". The perfect example of this is how when experiments came out seeming to indicate neutrinos were moving faster than light no serious scientists actually believed the neutrinos moved faster than light, rather, all the scientists rightly believed that there must have been something wrong with the experiment.



Now to answer your questions.





  1. Are any of Newton's three laws considered to be 'falsified theories' by any 'working physicists'? If so, what evidence do they have that they believe falsifies those three theories?




Despite what I said above the answer to your question is yes. This is because 'working physicists' are generally not good philosophers of science and many 'working physicists' incorrectly think Poppers program of falsification is correct. Working physicists aren't good philosophers of science because philosophy of science doesn't really help them do their job better and they simply may not find it that interesting, so if they hold misconceptions about philosophy of science it doesn't cause any problem whatsoever in their daily work.





  1. If the three laws are still unfalsified, are there any other concepts that form a part of "Newtonian Mechanics" that we consider to be falsified?




No. As I said above no physical theory can be falsified.



Here's some information about Imre Lakatos who has a better philosophy of science in my opinion than Popper.






share|cite|improve this answer









$endgroup$









  • 3




    $begingroup$
    I carry no water for Popper, but a statement like "no scientific theory is ever falsified" does not seem to me to be an actual description of what scientists think and do (as you have noted), and I question a theory of science that says that most scientists don't do science.
    $endgroup$
    – Rococo
    Apr 13 at 19:40






  • 2




    $begingroup$
    @Rococo I won't make any claim as to what percentage of scientists will say falsification is how science works because I've never done or seen a survey. I will point out that one can be a productive scientists regardless of ones opinions about philosophy of science. I would argue that all good scientists (even those who believe in falsification) do not actually do their science by trying to falsify things. Instead they do what all good scientists do:
    $endgroup$
    – jgerber
    Apr 13 at 21:37






  • 1




    $begingroup$
    They mess around with their theories and experiments seeing which theories work under which conditions, they try to figure out why and when theories break down, they generate new theories if necessary. The question is: does this theory explain what I am seeing? Why? Why not? What does it mean if this other theory also describes what I am seeing?
    $endgroup$
    – jgerber
    Apr 13 at 21:40






  • 1




    $begingroup$
    No where did I say that most scientists don't do science. All I said was that many scientists have misconceptions about philosophy of science but I was careful to point out that a scientists thoughts on philosophy of science have very little bearing on his or her aptitude as a scientist.
    $endgroup$
    – jgerber
    Apr 13 at 21:42










  • $begingroup$
    I apologize for mischaracterizing your position. Nonetheless, I am not personally convinced that, for example, 'all good scientists (even those who believe in falsification) do not actually do their science by trying to falsify things.' To be clear, I would neither make the extreme opposing claim that science is all about falsifying theories.
    $endgroup$
    – Rococo
    Apr 13 at 22:56



















1












$begingroup$

No, they are not considered falsified. They are still a valid low energy approximation, which is all they ever were claimed to be. Or, if you must, they have been falsified at very high energies only.






share|cite|improve this answer









$endgroup$









  • 2




    $begingroup$
    Did Newton claim his laws were a low energy approximation? I doubt it.
    $endgroup$
    – immibis
    2 days ago










  • $begingroup$
    @immibis he claimed his laws explained the solar system. That claim has not been falsified apart from extremely tiny deviations.
    $endgroup$
    – my2cts
    2 days ago








  • 1




    $begingroup$
    @my2cts Newtonian mechanics wasn't created to work "apart from extremely tiny derivations". It was created and applied to work at all scales. That's why we had to develop beyond classical/Newtonian mechanics when the model was no longer accurate. They were not as widely applicable as was originally thought.
    $endgroup$
    – JMac
    2 days ago










  • $begingroup$
    @JMac "as originally thought" Can you tell where in "Opticks" speeds of the order of 300.000 km/s are considered? Where an accuracy of 43 arcsec per century was claimed?
    $endgroup$
    – my2cts
    yesterday










  • $begingroup$
    @my2cts Can you show where he mentions that it's only exact when velocity is zero? I don't believe he states anywhere the assumptions that this doesn't apply at particular scales, or anything of the sort. All the evidence suggests that the physical laws he derived were to apply at all scales and velocities, and none of what he wrote that I'm aware of ever tried to mention that it was only an approximation for low energy.
    $endgroup$
    – JMac
    yesterday












Your Answer








StackExchange.ready(function() {
var channelOptions = {
tags: "".split(" "),
id: "151"
};
initTagRenderer("".split(" "), "".split(" "), channelOptions);

StackExchange.using("externalEditor", function() {
// Have to fire editor after snippets, if snippets enabled
if (StackExchange.settings.snippets.snippetsEnabled) {
StackExchange.using("snippets", function() {
createEditor();
});
}
else {
createEditor();
}
});

function createEditor() {
StackExchange.prepareEditor({
heartbeatType: 'answer',
autoActivateHeartbeat: false,
convertImagesToLinks: false,
noModals: true,
showLowRepImageUploadWarning: true,
reputationToPostImages: null,
bindNavPrevention: true,
postfix: "",
imageUploader: {
brandingHtml: "Powered by u003ca class="icon-imgur-white" href="https://imgur.com/"u003eu003c/au003e",
contentPolicyHtml: "User contributions licensed under u003ca href="https://creativecommons.org/licenses/by-sa/3.0/"u003ecc by-sa 3.0 with attribution requiredu003c/au003e u003ca href="https://stackoverflow.com/legal/content-policy"u003e(content policy)u003c/au003e",
allowUrls: true
},
noCode: true, onDemand: true,
discardSelector: ".discard-answer"
,immediatelyShowMarkdownHelp:true
});


}
});






Pod is a new contributor. Be nice, and check out our Code of Conduct.










draft saved

draft discarded


















StackExchange.ready(
function () {
StackExchange.openid.initPostLogin('.new-post-login', 'https%3a%2f%2fphysics.stackexchange.com%2fquestions%2f472215%2fdo-working-physicists-consider-newtonian-mechanics-to-be-falsified%23new-answer', 'question_page');
}
);

Post as a guest















Required, but never shown

























6 Answers
6






active

oldest

votes








6 Answers
6






active

oldest

votes









active

oldest

votes






active

oldest

votes









45












$begingroup$

"Falsified" is more philosophical than scientific distinction.
Newton laws have been falsified somehow, but we still use them, since usually they are a good approximation, and are easier to use than relativity or quantum mechanics.



The "action at distance" of Newton potentials has been falsified (finite speed of light...) but again, we use it every day.



So, in practical terms, no, Newton laws are still not falsified, in the sense that are not totally discredited in the scientific community. Classical mechanics is still in the curriculum of all universities, in a form more or less identical that 200 years ago (Before Relativity, quantum mechanics, field theory).



Most concept in physics fit more in the category of "methods" rather than "paradigms", so can be used over and over again. And all current methods and laws fails and give "false" results, when used outside their range of applicability.



The typical example of "falsified" theory is the Ptolemaic system of Sun & planets rotating around the Earth. However, philosopher usually omits the facts that:




  • Ptolemaic system was experimentally pretty good at calculating planet motions

  • Most mathematical and experimental methods of the new Heliocentric paradigm are the same of the old Ptolemaic


So the falsification was more on the point of view, rather than in the methods.






share|cite|improve this answer











$endgroup$









  • 3




    $begingroup$
    Well, we can measure that the Earth is rotating, against Ptolemaic system. But yes, we can still write ( with a lot of patience) all physics from our rotating system, with a lot of "fictitious" forces.
    $endgroup$
    – patta
    Apr 12 at 9:30








  • 20




    $begingroup$
    Note that both general mechanics and quantum mechanics are just as false as Newtonian mechanics, in a certain sense. QM fails to predict gravitational lensing, and GR fails to predict interference patterns in the double slit experiment. Both fail to explain how black holes preserve information. (one says they don't, the other says they don't exist)
    $endgroup$
    – John Dvorak
    Apr 12 at 10:04








  • 17




    $begingroup$
    @piet.t That's a common misconception. Ptolemy's model actually makes physically different predictions. Note that Venus is always between the Earth and the Sun in the Ptolemaic model, but not in reality. Galileo's observation of the phases of Venus, showing Venus fully illuminated by the sun, falsified the Ptolemaic model.
    $endgroup$
    – Denziloe
    Apr 12 at 11:08






  • 8




    $begingroup$
    99 c = 1 $ $pm$ 2%
    $endgroup$
    – patta
    Apr 12 at 18:41








  • 4




    $begingroup$
    @piet.t, the Ptolemaic system predicts that the fixed stars remain in the same relative locations at all times. Heliocentric models predict that they'll show annual variations in their positions relative to each other. High-precision observations of stellar positions show both parallax and aberration of light, which rather falsifies the Ptolemaic system (and almost all other geocentric systems).
    $endgroup$
    – Mark
    Apr 12 at 20:47


















45












$begingroup$

"Falsified" is more philosophical than scientific distinction.
Newton laws have been falsified somehow, but we still use them, since usually they are a good approximation, and are easier to use than relativity or quantum mechanics.



The "action at distance" of Newton potentials has been falsified (finite speed of light...) but again, we use it every day.



So, in practical terms, no, Newton laws are still not falsified, in the sense that are not totally discredited in the scientific community. Classical mechanics is still in the curriculum of all universities, in a form more or less identical that 200 years ago (Before Relativity, quantum mechanics, field theory).



Most concept in physics fit more in the category of "methods" rather than "paradigms", so can be used over and over again. And all current methods and laws fails and give "false" results, when used outside their range of applicability.



The typical example of "falsified" theory is the Ptolemaic system of Sun & planets rotating around the Earth. However, philosopher usually omits the facts that:




  • Ptolemaic system was experimentally pretty good at calculating planet motions

  • Most mathematical and experimental methods of the new Heliocentric paradigm are the same of the old Ptolemaic


So the falsification was more on the point of view, rather than in the methods.






share|cite|improve this answer











$endgroup$









  • 3




    $begingroup$
    Well, we can measure that the Earth is rotating, against Ptolemaic system. But yes, we can still write ( with a lot of patience) all physics from our rotating system, with a lot of "fictitious" forces.
    $endgroup$
    – patta
    Apr 12 at 9:30








  • 20




    $begingroup$
    Note that both general mechanics and quantum mechanics are just as false as Newtonian mechanics, in a certain sense. QM fails to predict gravitational lensing, and GR fails to predict interference patterns in the double slit experiment. Both fail to explain how black holes preserve information. (one says they don't, the other says they don't exist)
    $endgroup$
    – John Dvorak
    Apr 12 at 10:04








  • 17




    $begingroup$
    @piet.t That's a common misconception. Ptolemy's model actually makes physically different predictions. Note that Venus is always between the Earth and the Sun in the Ptolemaic model, but not in reality. Galileo's observation of the phases of Venus, showing Venus fully illuminated by the sun, falsified the Ptolemaic model.
    $endgroup$
    – Denziloe
    Apr 12 at 11:08






  • 8




    $begingroup$
    99 c = 1 $ $pm$ 2%
    $endgroup$
    – patta
    Apr 12 at 18:41








  • 4




    $begingroup$
    @piet.t, the Ptolemaic system predicts that the fixed stars remain in the same relative locations at all times. Heliocentric models predict that they'll show annual variations in their positions relative to each other. High-precision observations of stellar positions show both parallax and aberration of light, which rather falsifies the Ptolemaic system (and almost all other geocentric systems).
    $endgroup$
    – Mark
    Apr 12 at 20:47
















45












45








45





$begingroup$

"Falsified" is more philosophical than scientific distinction.
Newton laws have been falsified somehow, but we still use them, since usually they are a good approximation, and are easier to use than relativity or quantum mechanics.



The "action at distance" of Newton potentials has been falsified (finite speed of light...) but again, we use it every day.



So, in practical terms, no, Newton laws are still not falsified, in the sense that are not totally discredited in the scientific community. Classical mechanics is still in the curriculum of all universities, in a form more or less identical that 200 years ago (Before Relativity, quantum mechanics, field theory).



Most concept in physics fit more in the category of "methods" rather than "paradigms", so can be used over and over again. And all current methods and laws fails and give "false" results, when used outside their range of applicability.



The typical example of "falsified" theory is the Ptolemaic system of Sun & planets rotating around the Earth. However, philosopher usually omits the facts that:




  • Ptolemaic system was experimentally pretty good at calculating planet motions

  • Most mathematical and experimental methods of the new Heliocentric paradigm are the same of the old Ptolemaic


So the falsification was more on the point of view, rather than in the methods.






share|cite|improve this answer











$endgroup$



"Falsified" is more philosophical than scientific distinction.
Newton laws have been falsified somehow, but we still use them, since usually they are a good approximation, and are easier to use than relativity or quantum mechanics.



The "action at distance" of Newton potentials has been falsified (finite speed of light...) but again, we use it every day.



So, in practical terms, no, Newton laws are still not falsified, in the sense that are not totally discredited in the scientific community. Classical mechanics is still in the curriculum of all universities, in a form more or less identical that 200 years ago (Before Relativity, quantum mechanics, field theory).



Most concept in physics fit more in the category of "methods" rather than "paradigms", so can be used over and over again. And all current methods and laws fails and give "false" results, when used outside their range of applicability.



The typical example of "falsified" theory is the Ptolemaic system of Sun & planets rotating around the Earth. However, philosopher usually omits the facts that:




  • Ptolemaic system was experimentally pretty good at calculating planet motions

  • Most mathematical and experimental methods of the new Heliocentric paradigm are the same of the old Ptolemaic


So the falsification was more on the point of view, rather than in the methods.







share|cite|improve this answer














share|cite|improve this answer



share|cite|improve this answer








edited Apr 12 at 8:36

























answered Apr 12 at 8:21









pattapatta

76648




76648








  • 3




    $begingroup$
    Well, we can measure that the Earth is rotating, against Ptolemaic system. But yes, we can still write ( with a lot of patience) all physics from our rotating system, with a lot of "fictitious" forces.
    $endgroup$
    – patta
    Apr 12 at 9:30








  • 20




    $begingroup$
    Note that both general mechanics and quantum mechanics are just as false as Newtonian mechanics, in a certain sense. QM fails to predict gravitational lensing, and GR fails to predict interference patterns in the double slit experiment. Both fail to explain how black holes preserve information. (one says they don't, the other says they don't exist)
    $endgroup$
    – John Dvorak
    Apr 12 at 10:04








  • 17




    $begingroup$
    @piet.t That's a common misconception. Ptolemy's model actually makes physically different predictions. Note that Venus is always between the Earth and the Sun in the Ptolemaic model, but not in reality. Galileo's observation of the phases of Venus, showing Venus fully illuminated by the sun, falsified the Ptolemaic model.
    $endgroup$
    – Denziloe
    Apr 12 at 11:08






  • 8




    $begingroup$
    99 c = 1 $ $pm$ 2%
    $endgroup$
    – patta
    Apr 12 at 18:41








  • 4




    $begingroup$
    @piet.t, the Ptolemaic system predicts that the fixed stars remain in the same relative locations at all times. Heliocentric models predict that they'll show annual variations in their positions relative to each other. High-precision observations of stellar positions show both parallax and aberration of light, which rather falsifies the Ptolemaic system (and almost all other geocentric systems).
    $endgroup$
    – Mark
    Apr 12 at 20:47
















  • 3




    $begingroup$
    Well, we can measure that the Earth is rotating, against Ptolemaic system. But yes, we can still write ( with a lot of patience) all physics from our rotating system, with a lot of "fictitious" forces.
    $endgroup$
    – patta
    Apr 12 at 9:30








  • 20




    $begingroup$
    Note that both general mechanics and quantum mechanics are just as false as Newtonian mechanics, in a certain sense. QM fails to predict gravitational lensing, and GR fails to predict interference patterns in the double slit experiment. Both fail to explain how black holes preserve information. (one says they don't, the other says they don't exist)
    $endgroup$
    – John Dvorak
    Apr 12 at 10:04








  • 17




    $begingroup$
    @piet.t That's a common misconception. Ptolemy's model actually makes physically different predictions. Note that Venus is always between the Earth and the Sun in the Ptolemaic model, but not in reality. Galileo's observation of the phases of Venus, showing Venus fully illuminated by the sun, falsified the Ptolemaic model.
    $endgroup$
    – Denziloe
    Apr 12 at 11:08






  • 8




    $begingroup$
    99 c = 1 $ $pm$ 2%
    $endgroup$
    – patta
    Apr 12 at 18:41








  • 4




    $begingroup$
    @piet.t, the Ptolemaic system predicts that the fixed stars remain in the same relative locations at all times. Heliocentric models predict that they'll show annual variations in their positions relative to each other. High-precision observations of stellar positions show both parallax and aberration of light, which rather falsifies the Ptolemaic system (and almost all other geocentric systems).
    $endgroup$
    – Mark
    Apr 12 at 20:47










3




3




$begingroup$
Well, we can measure that the Earth is rotating, against Ptolemaic system. But yes, we can still write ( with a lot of patience) all physics from our rotating system, with a lot of "fictitious" forces.
$endgroup$
– patta
Apr 12 at 9:30






$begingroup$
Well, we can measure that the Earth is rotating, against Ptolemaic system. But yes, we can still write ( with a lot of patience) all physics from our rotating system, with a lot of "fictitious" forces.
$endgroup$
– patta
Apr 12 at 9:30






20




20




$begingroup$
Note that both general mechanics and quantum mechanics are just as false as Newtonian mechanics, in a certain sense. QM fails to predict gravitational lensing, and GR fails to predict interference patterns in the double slit experiment. Both fail to explain how black holes preserve information. (one says they don't, the other says they don't exist)
$endgroup$
– John Dvorak
Apr 12 at 10:04






$begingroup$
Note that both general mechanics and quantum mechanics are just as false as Newtonian mechanics, in a certain sense. QM fails to predict gravitational lensing, and GR fails to predict interference patterns in the double slit experiment. Both fail to explain how black holes preserve information. (one says they don't, the other says they don't exist)
$endgroup$
– John Dvorak
Apr 12 at 10:04






17




17




$begingroup$
@piet.t That's a common misconception. Ptolemy's model actually makes physically different predictions. Note that Venus is always between the Earth and the Sun in the Ptolemaic model, but not in reality. Galileo's observation of the phases of Venus, showing Venus fully illuminated by the sun, falsified the Ptolemaic model.
$endgroup$
– Denziloe
Apr 12 at 11:08




$begingroup$
@piet.t That's a common misconception. Ptolemy's model actually makes physically different predictions. Note that Venus is always between the Earth and the Sun in the Ptolemaic model, but not in reality. Galileo's observation of the phases of Venus, showing Venus fully illuminated by the sun, falsified the Ptolemaic model.
$endgroup$
– Denziloe
Apr 12 at 11:08




8




8




$begingroup$
99 c = 1 $ $pm$ 2%
$endgroup$
– patta
Apr 12 at 18:41






$begingroup$
99 c = 1 $ $pm$ 2%
$endgroup$
– patta
Apr 12 at 18:41






4




4




$begingroup$
@piet.t, the Ptolemaic system predicts that the fixed stars remain in the same relative locations at all times. Heliocentric models predict that they'll show annual variations in their positions relative to each other. High-precision observations of stellar positions show both parallax and aberration of light, which rather falsifies the Ptolemaic system (and almost all other geocentric systems).
$endgroup$
– Mark
Apr 12 at 20:47






$begingroup$
@piet.t, the Ptolemaic system predicts that the fixed stars remain in the same relative locations at all times. Heliocentric models predict that they'll show annual variations in their positions relative to each other. High-precision observations of stellar positions show both parallax and aberration of light, which rather falsifies the Ptolemaic system (and almost all other geocentric systems).
$endgroup$
– Mark
Apr 12 at 20:47













29












$begingroup$

Newtonian Physics is accurate in the specific domain it was designed for



Physics is not about identifying the "truth" of the world around us. It's about creating mathematical models that allow us to accurately predict the behavior of the world.



Nobody is trying to create a perfect model, because the complexity of such a model would be infinite. Instead, we look for the boundaries of a model's accuracy - under what conditions it produces reasonable results, and the precision of the results it produces under those conditions.



You can see this more clearly with other physics models, such as the Ideal Gas Law. The Ideal Gas Law models a hugely complex system of particle collisions as a simple formula of ratios. It breaks down relatively quickly at high or low values of any of its quantities, but because we understand when and how the law breaks down, it is still useful.



At extremely large quantities (large speeds, large masses, high energies), the Newtonian model starts to break down, and we need to use a Relativistic model in order to get accurate results. But that doesn't mean that the Newtonian model is false, it just means that it is inapplicable for those conditions.



Obviously, Newton wasn't aware of the limitations to his laws when he described them. He was trying to create a universally applicable set of relations. In that sense you could argue that he failed. But I would consider the modern understanding a refinement of his laws, rather than a falsification.






share|cite|improve this answer











$endgroup$









  • 1




    $begingroup$
    Avoids answering the question...
    $endgroup$
    – Rob Jeffries
    Apr 13 at 16:53






  • 6




    $begingroup$
    This is the modern understanding, but I don't think it is true to say that Newton's laws were designed (by Newton, at least) for slow speeds and mild gravitational curvatures.
    $endgroup$
    – Rococo
    Apr 13 at 19:36






  • 5




    $begingroup$
    This answer would probably make more sense if "designed for" was replaced with "conceptualised within".
    $endgroup$
    – Ian Kemp
    Apr 13 at 22:37






  • 2




    $begingroup$
    @IanKemp I would've awarded a bounty on this comment if it were a feature! I mean, of course, Newtonian mechanics was designed for everything. It was just conceptualized within a certain regime of experiments. And it is not in the spirit of science to actually design a theory for the regime in which the experimental results are already known. A scientific theory has to make predictions and that means that it necessarily has to go beyond the domain from which it takes empirical inspiration.
    $endgroup$
    – Dvij Mankad
    Apr 13 at 23:33










  • $begingroup$
    When Newton was studying Kepler's work, he probably wasn't wondering if he could apply it to the $1s$ orbital of the hydrogen atom.
    $endgroup$
    – Cinaed Simson
    2 days ago


















29












$begingroup$

Newtonian Physics is accurate in the specific domain it was designed for



Physics is not about identifying the "truth" of the world around us. It's about creating mathematical models that allow us to accurately predict the behavior of the world.



Nobody is trying to create a perfect model, because the complexity of such a model would be infinite. Instead, we look for the boundaries of a model's accuracy - under what conditions it produces reasonable results, and the precision of the results it produces under those conditions.



You can see this more clearly with other physics models, such as the Ideal Gas Law. The Ideal Gas Law models a hugely complex system of particle collisions as a simple formula of ratios. It breaks down relatively quickly at high or low values of any of its quantities, but because we understand when and how the law breaks down, it is still useful.



At extremely large quantities (large speeds, large masses, high energies), the Newtonian model starts to break down, and we need to use a Relativistic model in order to get accurate results. But that doesn't mean that the Newtonian model is false, it just means that it is inapplicable for those conditions.



Obviously, Newton wasn't aware of the limitations to his laws when he described them. He was trying to create a universally applicable set of relations. In that sense you could argue that he failed. But I would consider the modern understanding a refinement of his laws, rather than a falsification.






share|cite|improve this answer











$endgroup$









  • 1




    $begingroup$
    Avoids answering the question...
    $endgroup$
    – Rob Jeffries
    Apr 13 at 16:53






  • 6




    $begingroup$
    This is the modern understanding, but I don't think it is true to say that Newton's laws were designed (by Newton, at least) for slow speeds and mild gravitational curvatures.
    $endgroup$
    – Rococo
    Apr 13 at 19:36






  • 5




    $begingroup$
    This answer would probably make more sense if "designed for" was replaced with "conceptualised within".
    $endgroup$
    – Ian Kemp
    Apr 13 at 22:37






  • 2




    $begingroup$
    @IanKemp I would've awarded a bounty on this comment if it were a feature! I mean, of course, Newtonian mechanics was designed for everything. It was just conceptualized within a certain regime of experiments. And it is not in the spirit of science to actually design a theory for the regime in which the experimental results are already known. A scientific theory has to make predictions and that means that it necessarily has to go beyond the domain from which it takes empirical inspiration.
    $endgroup$
    – Dvij Mankad
    Apr 13 at 23:33










  • $begingroup$
    When Newton was studying Kepler's work, he probably wasn't wondering if he could apply it to the $1s$ orbital of the hydrogen atom.
    $endgroup$
    – Cinaed Simson
    2 days ago
















29












29








29





$begingroup$

Newtonian Physics is accurate in the specific domain it was designed for



Physics is not about identifying the "truth" of the world around us. It's about creating mathematical models that allow us to accurately predict the behavior of the world.



Nobody is trying to create a perfect model, because the complexity of such a model would be infinite. Instead, we look for the boundaries of a model's accuracy - under what conditions it produces reasonable results, and the precision of the results it produces under those conditions.



You can see this more clearly with other physics models, such as the Ideal Gas Law. The Ideal Gas Law models a hugely complex system of particle collisions as a simple formula of ratios. It breaks down relatively quickly at high or low values of any of its quantities, but because we understand when and how the law breaks down, it is still useful.



At extremely large quantities (large speeds, large masses, high energies), the Newtonian model starts to break down, and we need to use a Relativistic model in order to get accurate results. But that doesn't mean that the Newtonian model is false, it just means that it is inapplicable for those conditions.



Obviously, Newton wasn't aware of the limitations to his laws when he described them. He was trying to create a universally applicable set of relations. In that sense you could argue that he failed. But I would consider the modern understanding a refinement of his laws, rather than a falsification.






share|cite|improve this answer











$endgroup$



Newtonian Physics is accurate in the specific domain it was designed for



Physics is not about identifying the "truth" of the world around us. It's about creating mathematical models that allow us to accurately predict the behavior of the world.



Nobody is trying to create a perfect model, because the complexity of such a model would be infinite. Instead, we look for the boundaries of a model's accuracy - under what conditions it produces reasonable results, and the precision of the results it produces under those conditions.



You can see this more clearly with other physics models, such as the Ideal Gas Law. The Ideal Gas Law models a hugely complex system of particle collisions as a simple formula of ratios. It breaks down relatively quickly at high or low values of any of its quantities, but because we understand when and how the law breaks down, it is still useful.



At extremely large quantities (large speeds, large masses, high energies), the Newtonian model starts to break down, and we need to use a Relativistic model in order to get accurate results. But that doesn't mean that the Newtonian model is false, it just means that it is inapplicable for those conditions.



Obviously, Newton wasn't aware of the limitations to his laws when he described them. He was trying to create a universally applicable set of relations. In that sense you could argue that he failed. But I would consider the modern understanding a refinement of his laws, rather than a falsification.







share|cite|improve this answer














share|cite|improve this answer



share|cite|improve this answer








edited Apr 14 at 18:01

























answered Apr 12 at 18:32









Arcanist LupusArcanist Lupus

41724




41724








  • 1




    $begingroup$
    Avoids answering the question...
    $endgroup$
    – Rob Jeffries
    Apr 13 at 16:53






  • 6




    $begingroup$
    This is the modern understanding, but I don't think it is true to say that Newton's laws were designed (by Newton, at least) for slow speeds and mild gravitational curvatures.
    $endgroup$
    – Rococo
    Apr 13 at 19:36






  • 5




    $begingroup$
    This answer would probably make more sense if "designed for" was replaced with "conceptualised within".
    $endgroup$
    – Ian Kemp
    Apr 13 at 22:37






  • 2




    $begingroup$
    @IanKemp I would've awarded a bounty on this comment if it were a feature! I mean, of course, Newtonian mechanics was designed for everything. It was just conceptualized within a certain regime of experiments. And it is not in the spirit of science to actually design a theory for the regime in which the experimental results are already known. A scientific theory has to make predictions and that means that it necessarily has to go beyond the domain from which it takes empirical inspiration.
    $endgroup$
    – Dvij Mankad
    Apr 13 at 23:33










  • $begingroup$
    When Newton was studying Kepler's work, he probably wasn't wondering if he could apply it to the $1s$ orbital of the hydrogen atom.
    $endgroup$
    – Cinaed Simson
    2 days ago
















  • 1




    $begingroup$
    Avoids answering the question...
    $endgroup$
    – Rob Jeffries
    Apr 13 at 16:53






  • 6




    $begingroup$
    This is the modern understanding, but I don't think it is true to say that Newton's laws were designed (by Newton, at least) for slow speeds and mild gravitational curvatures.
    $endgroup$
    – Rococo
    Apr 13 at 19:36






  • 5




    $begingroup$
    This answer would probably make more sense if "designed for" was replaced with "conceptualised within".
    $endgroup$
    – Ian Kemp
    Apr 13 at 22:37






  • 2




    $begingroup$
    @IanKemp I would've awarded a bounty on this comment if it were a feature! I mean, of course, Newtonian mechanics was designed for everything. It was just conceptualized within a certain regime of experiments. And it is not in the spirit of science to actually design a theory for the regime in which the experimental results are already known. A scientific theory has to make predictions and that means that it necessarily has to go beyond the domain from which it takes empirical inspiration.
    $endgroup$
    – Dvij Mankad
    Apr 13 at 23:33










  • $begingroup$
    When Newton was studying Kepler's work, he probably wasn't wondering if he could apply it to the $1s$ orbital of the hydrogen atom.
    $endgroup$
    – Cinaed Simson
    2 days ago










1




1




$begingroup$
Avoids answering the question...
$endgroup$
– Rob Jeffries
Apr 13 at 16:53




$begingroup$
Avoids answering the question...
$endgroup$
– Rob Jeffries
Apr 13 at 16:53




6




6




$begingroup$
This is the modern understanding, but I don't think it is true to say that Newton's laws were designed (by Newton, at least) for slow speeds and mild gravitational curvatures.
$endgroup$
– Rococo
Apr 13 at 19:36




$begingroup$
This is the modern understanding, but I don't think it is true to say that Newton's laws were designed (by Newton, at least) for slow speeds and mild gravitational curvatures.
$endgroup$
– Rococo
Apr 13 at 19:36




5




5




$begingroup$
This answer would probably make more sense if "designed for" was replaced with "conceptualised within".
$endgroup$
– Ian Kemp
Apr 13 at 22:37




$begingroup$
This answer would probably make more sense if "designed for" was replaced with "conceptualised within".
$endgroup$
– Ian Kemp
Apr 13 at 22:37




2




2




$begingroup$
@IanKemp I would've awarded a bounty on this comment if it were a feature! I mean, of course, Newtonian mechanics was designed for everything. It was just conceptualized within a certain regime of experiments. And it is not in the spirit of science to actually design a theory for the regime in which the experimental results are already known. A scientific theory has to make predictions and that means that it necessarily has to go beyond the domain from which it takes empirical inspiration.
$endgroup$
– Dvij Mankad
Apr 13 at 23:33




$begingroup$
@IanKemp I would've awarded a bounty on this comment if it were a feature! I mean, of course, Newtonian mechanics was designed for everything. It was just conceptualized within a certain regime of experiments. And it is not in the spirit of science to actually design a theory for the regime in which the experimental results are already known. A scientific theory has to make predictions and that means that it necessarily has to go beyond the domain from which it takes empirical inspiration.
$endgroup$
– Dvij Mankad
Apr 13 at 23:33












$begingroup$
When Newton was studying Kepler's work, he probably wasn't wondering if he could apply it to the $1s$ orbital of the hydrogen atom.
$endgroup$
– Cinaed Simson
2 days ago






$begingroup$
When Newton was studying Kepler's work, he probably wasn't wondering if he could apply it to the $1s$ orbital of the hydrogen atom.
$endgroup$
– Cinaed Simson
2 days ago













9












$begingroup$

One of the problems of Newton's law of universal gravitation, $$F_text{Grav} = G frac{m_1m_2}{r^2},$$ is that it does not correctly describe the precession of Mercury's orbit. Mercury behaves slightly different than predicted by Newton's law and general relativity does a better job.



See also the corresponding Wikipedia article.






share|cite|improve this answer











$endgroup$









  • 2




    $begingroup$
    When you say "Newton's Laws", which do you mean exactly? There are laws for inertial motion, action/reaction, force as dp/dt, and gravity. I believe only the last one could be seen as needing modification by General Relativity.
    $endgroup$
    – Jens
    Apr 13 at 10:55










  • $begingroup$
    Thanks, clarified.
    $endgroup$
    – Jasper
    Apr 13 at 16:49










  • $begingroup$
    Indeed Mercury's elliptical orbit slowly rotates by a tiny amount extra than what Newton's gravity predicts. However people generally underemphasise other precession effects: 5000 "/c [seconds of arc per century] from precession of the equinoxes, and 530 "/c from other planets, compared with the observed 43 "/c extra that general relativity (and other gravity theories) explains. So Newtonian gravity is correct to within less than 1% error, in explaining the precession of Mercury's orbit.
    $endgroup$
    – Colin MacLaurin
    yesterday
















9












$begingroup$

One of the problems of Newton's law of universal gravitation, $$F_text{Grav} = G frac{m_1m_2}{r^2},$$ is that it does not correctly describe the precession of Mercury's orbit. Mercury behaves slightly different than predicted by Newton's law and general relativity does a better job.



See also the corresponding Wikipedia article.






share|cite|improve this answer











$endgroup$









  • 2




    $begingroup$
    When you say "Newton's Laws", which do you mean exactly? There are laws for inertial motion, action/reaction, force as dp/dt, and gravity. I believe only the last one could be seen as needing modification by General Relativity.
    $endgroup$
    – Jens
    Apr 13 at 10:55










  • $begingroup$
    Thanks, clarified.
    $endgroup$
    – Jasper
    Apr 13 at 16:49










  • $begingroup$
    Indeed Mercury's elliptical orbit slowly rotates by a tiny amount extra than what Newton's gravity predicts. However people generally underemphasise other precession effects: 5000 "/c [seconds of arc per century] from precession of the equinoxes, and 530 "/c from other planets, compared with the observed 43 "/c extra that general relativity (and other gravity theories) explains. So Newtonian gravity is correct to within less than 1% error, in explaining the precession of Mercury's orbit.
    $endgroup$
    – Colin MacLaurin
    yesterday














9












9








9





$begingroup$

One of the problems of Newton's law of universal gravitation, $$F_text{Grav} = G frac{m_1m_2}{r^2},$$ is that it does not correctly describe the precession of Mercury's orbit. Mercury behaves slightly different than predicted by Newton's law and general relativity does a better job.



See also the corresponding Wikipedia article.






share|cite|improve this answer











$endgroup$



One of the problems of Newton's law of universal gravitation, $$F_text{Grav} = G frac{m_1m_2}{r^2},$$ is that it does not correctly describe the precession of Mercury's orbit. Mercury behaves slightly different than predicted by Newton's law and general relativity does a better job.



See also the corresponding Wikipedia article.







share|cite|improve this answer














share|cite|improve this answer



share|cite|improve this answer








edited Apr 13 at 16:49

























answered Apr 12 at 8:26









JasperJasper

1,2521517




1,2521517








  • 2




    $begingroup$
    When you say "Newton's Laws", which do you mean exactly? There are laws for inertial motion, action/reaction, force as dp/dt, and gravity. I believe only the last one could be seen as needing modification by General Relativity.
    $endgroup$
    – Jens
    Apr 13 at 10:55










  • $begingroup$
    Thanks, clarified.
    $endgroup$
    – Jasper
    Apr 13 at 16:49










  • $begingroup$
    Indeed Mercury's elliptical orbit slowly rotates by a tiny amount extra than what Newton's gravity predicts. However people generally underemphasise other precession effects: 5000 "/c [seconds of arc per century] from precession of the equinoxes, and 530 "/c from other planets, compared with the observed 43 "/c extra that general relativity (and other gravity theories) explains. So Newtonian gravity is correct to within less than 1% error, in explaining the precession of Mercury's orbit.
    $endgroup$
    – Colin MacLaurin
    yesterday














  • 2




    $begingroup$
    When you say "Newton's Laws", which do you mean exactly? There are laws for inertial motion, action/reaction, force as dp/dt, and gravity. I believe only the last one could be seen as needing modification by General Relativity.
    $endgroup$
    – Jens
    Apr 13 at 10:55










  • $begingroup$
    Thanks, clarified.
    $endgroup$
    – Jasper
    Apr 13 at 16:49










  • $begingroup$
    Indeed Mercury's elliptical orbit slowly rotates by a tiny amount extra than what Newton's gravity predicts. However people generally underemphasise other precession effects: 5000 "/c [seconds of arc per century] from precession of the equinoxes, and 530 "/c from other planets, compared with the observed 43 "/c extra that general relativity (and other gravity theories) explains. So Newtonian gravity is correct to within less than 1% error, in explaining the precession of Mercury's orbit.
    $endgroup$
    – Colin MacLaurin
    yesterday








2




2




$begingroup$
When you say "Newton's Laws", which do you mean exactly? There are laws for inertial motion, action/reaction, force as dp/dt, and gravity. I believe only the last one could be seen as needing modification by General Relativity.
$endgroup$
– Jens
Apr 13 at 10:55




$begingroup$
When you say "Newton's Laws", which do you mean exactly? There are laws for inertial motion, action/reaction, force as dp/dt, and gravity. I believe only the last one could be seen as needing modification by General Relativity.
$endgroup$
– Jens
Apr 13 at 10:55












$begingroup$
Thanks, clarified.
$endgroup$
– Jasper
Apr 13 at 16:49




$begingroup$
Thanks, clarified.
$endgroup$
– Jasper
Apr 13 at 16:49












$begingroup$
Indeed Mercury's elliptical orbit slowly rotates by a tiny amount extra than what Newton's gravity predicts. However people generally underemphasise other precession effects: 5000 "/c [seconds of arc per century] from precession of the equinoxes, and 530 "/c from other planets, compared with the observed 43 "/c extra that general relativity (and other gravity theories) explains. So Newtonian gravity is correct to within less than 1% error, in explaining the precession of Mercury's orbit.
$endgroup$
– Colin MacLaurin
yesterday




$begingroup$
Indeed Mercury's elliptical orbit slowly rotates by a tiny amount extra than what Newton's gravity predicts. However people generally underemphasise other precession effects: 5000 "/c [seconds of arc per century] from precession of the equinoxes, and 530 "/c from other planets, compared with the observed 43 "/c extra that general relativity (and other gravity theories) explains. So Newtonian gravity is correct to within less than 1% error, in explaining the precession of Mercury's orbit.
$endgroup$
– Colin MacLaurin
yesterday











2












$begingroup$

Relativity is an extension of Newtonian physics, not either a replacement or correction. As such, relativity does not "falsify" Newtonian physics. For velocities far smaller than the speed of light (approaching zero), relativity simplifies back to the Newtonian model. For everyday use, and for everyday engineering problems, Newtonian physics is more than accurate enough. It's only when you get into more "interesting" situations that Newtonian physics fails to provide adequate solutions. The orbit of Mercury is a famous one. It's only because of Mercury's proximity to the Sun that its orbit defies accurate modeling in purely Newtonian terms. Similarly, without an understanding of relativity and relativistic effects on orbiting spacecraft, the GPS system could not work (the onboard timekeeping of the GPS satellites must be extremely precise and the very small relativisitic effects on their clocks must be accounted for). These are not everyday situations, and the relativistic effects are small, but the position of Mercury can be very precisely measured and GPS signals are timed with very high precision (light/radio travels about a foot or about 30cm in a nanosecond).






share|cite|improve this answer









$endgroup$













  • $begingroup$
    This answer seems to flip between two contradictory viewpoints. You say relativity is an "extension" of Newtonian physics, not a replacement or correction; but then proceed to talk about Newtonian physics "failing to provide adequate solutions" unless relativity is accounted for. Wouldn't that make it a "correction" to Newtonian physics in most senses of the term? I wouldn't consider new information that makes all prior information slightly inaccurate an "extension"; I would specifically call it a correction; because it corrects the errors in Newtonian physics.
    $endgroup$
    – JMac
    2 days ago










  • $begingroup$
    @JMac I stand by my core point that relativity is an extension not a correction because of the way the relativistic terms cancel out (leaving the simplified Newtonian forms) as velocity approaches zero (or the comparable terms which relate to curvature of spacetime due to mass).
    $endgroup$
    – Anthony X
    2 days ago


















2












$begingroup$

Relativity is an extension of Newtonian physics, not either a replacement or correction. As such, relativity does not "falsify" Newtonian physics. For velocities far smaller than the speed of light (approaching zero), relativity simplifies back to the Newtonian model. For everyday use, and for everyday engineering problems, Newtonian physics is more than accurate enough. It's only when you get into more "interesting" situations that Newtonian physics fails to provide adequate solutions. The orbit of Mercury is a famous one. It's only because of Mercury's proximity to the Sun that its orbit defies accurate modeling in purely Newtonian terms. Similarly, without an understanding of relativity and relativistic effects on orbiting spacecraft, the GPS system could not work (the onboard timekeeping of the GPS satellites must be extremely precise and the very small relativisitic effects on their clocks must be accounted for). These are not everyday situations, and the relativistic effects are small, but the position of Mercury can be very precisely measured and GPS signals are timed with very high precision (light/radio travels about a foot or about 30cm in a nanosecond).






share|cite|improve this answer









$endgroup$













  • $begingroup$
    This answer seems to flip between two contradictory viewpoints. You say relativity is an "extension" of Newtonian physics, not a replacement or correction; but then proceed to talk about Newtonian physics "failing to provide adequate solutions" unless relativity is accounted for. Wouldn't that make it a "correction" to Newtonian physics in most senses of the term? I wouldn't consider new information that makes all prior information slightly inaccurate an "extension"; I would specifically call it a correction; because it corrects the errors in Newtonian physics.
    $endgroup$
    – JMac
    2 days ago










  • $begingroup$
    @JMac I stand by my core point that relativity is an extension not a correction because of the way the relativistic terms cancel out (leaving the simplified Newtonian forms) as velocity approaches zero (or the comparable terms which relate to curvature of spacetime due to mass).
    $endgroup$
    – Anthony X
    2 days ago
















2












2








2





$begingroup$

Relativity is an extension of Newtonian physics, not either a replacement or correction. As such, relativity does not "falsify" Newtonian physics. For velocities far smaller than the speed of light (approaching zero), relativity simplifies back to the Newtonian model. For everyday use, and for everyday engineering problems, Newtonian physics is more than accurate enough. It's only when you get into more "interesting" situations that Newtonian physics fails to provide adequate solutions. The orbit of Mercury is a famous one. It's only because of Mercury's proximity to the Sun that its orbit defies accurate modeling in purely Newtonian terms. Similarly, without an understanding of relativity and relativistic effects on orbiting spacecraft, the GPS system could not work (the onboard timekeeping of the GPS satellites must be extremely precise and the very small relativisitic effects on their clocks must be accounted for). These are not everyday situations, and the relativistic effects are small, but the position of Mercury can be very precisely measured and GPS signals are timed with very high precision (light/radio travels about a foot or about 30cm in a nanosecond).






share|cite|improve this answer









$endgroup$



Relativity is an extension of Newtonian physics, not either a replacement or correction. As such, relativity does not "falsify" Newtonian physics. For velocities far smaller than the speed of light (approaching zero), relativity simplifies back to the Newtonian model. For everyday use, and for everyday engineering problems, Newtonian physics is more than accurate enough. It's only when you get into more "interesting" situations that Newtonian physics fails to provide adequate solutions. The orbit of Mercury is a famous one. It's only because of Mercury's proximity to the Sun that its orbit defies accurate modeling in purely Newtonian terms. Similarly, without an understanding of relativity and relativistic effects on orbiting spacecraft, the GPS system could not work (the onboard timekeeping of the GPS satellites must be extremely precise and the very small relativisitic effects on their clocks must be accounted for). These are not everyday situations, and the relativistic effects are small, but the position of Mercury can be very precisely measured and GPS signals are timed with very high precision (light/radio travels about a foot or about 30cm in a nanosecond).







share|cite|improve this answer












share|cite|improve this answer



share|cite|improve this answer










answered Apr 14 at 22:11









Anthony XAnthony X

2,80611220




2,80611220












  • $begingroup$
    This answer seems to flip between two contradictory viewpoints. You say relativity is an "extension" of Newtonian physics, not a replacement or correction; but then proceed to talk about Newtonian physics "failing to provide adequate solutions" unless relativity is accounted for. Wouldn't that make it a "correction" to Newtonian physics in most senses of the term? I wouldn't consider new information that makes all prior information slightly inaccurate an "extension"; I would specifically call it a correction; because it corrects the errors in Newtonian physics.
    $endgroup$
    – JMac
    2 days ago










  • $begingroup$
    @JMac I stand by my core point that relativity is an extension not a correction because of the way the relativistic terms cancel out (leaving the simplified Newtonian forms) as velocity approaches zero (or the comparable terms which relate to curvature of spacetime due to mass).
    $endgroup$
    – Anthony X
    2 days ago




















  • $begingroup$
    This answer seems to flip between two contradictory viewpoints. You say relativity is an "extension" of Newtonian physics, not a replacement or correction; but then proceed to talk about Newtonian physics "failing to provide adequate solutions" unless relativity is accounted for. Wouldn't that make it a "correction" to Newtonian physics in most senses of the term? I wouldn't consider new information that makes all prior information slightly inaccurate an "extension"; I would specifically call it a correction; because it corrects the errors in Newtonian physics.
    $endgroup$
    – JMac
    2 days ago










  • $begingroup$
    @JMac I stand by my core point that relativity is an extension not a correction because of the way the relativistic terms cancel out (leaving the simplified Newtonian forms) as velocity approaches zero (or the comparable terms which relate to curvature of spacetime due to mass).
    $endgroup$
    – Anthony X
    2 days ago


















$begingroup$
This answer seems to flip between two contradictory viewpoints. You say relativity is an "extension" of Newtonian physics, not a replacement or correction; but then proceed to talk about Newtonian physics "failing to provide adequate solutions" unless relativity is accounted for. Wouldn't that make it a "correction" to Newtonian physics in most senses of the term? I wouldn't consider new information that makes all prior information slightly inaccurate an "extension"; I would specifically call it a correction; because it corrects the errors in Newtonian physics.
$endgroup$
– JMac
2 days ago




$begingroup$
This answer seems to flip between two contradictory viewpoints. You say relativity is an "extension" of Newtonian physics, not a replacement or correction; but then proceed to talk about Newtonian physics "failing to provide adequate solutions" unless relativity is accounted for. Wouldn't that make it a "correction" to Newtonian physics in most senses of the term? I wouldn't consider new information that makes all prior information slightly inaccurate an "extension"; I would specifically call it a correction; because it corrects the errors in Newtonian physics.
$endgroup$
– JMac
2 days ago












$begingroup$
@JMac I stand by my core point that relativity is an extension not a correction because of the way the relativistic terms cancel out (leaving the simplified Newtonian forms) as velocity approaches zero (or the comparable terms which relate to curvature of spacetime due to mass).
$endgroup$
– Anthony X
2 days ago






$begingroup$
@JMac I stand by my core point that relativity is an extension not a correction because of the way the relativistic terms cancel out (leaving the simplified Newtonian forms) as velocity approaches zero (or the comparable terms which relate to curvature of spacetime due to mass).
$endgroup$
– Anthony X
2 days ago













1












$begingroup$

First of all no scientific theory can possibly be falsified. Popper was wrong. See the Quine-Duhem thesis which says that instead of rejecting the theory when a seemingly falsifying experiment occurs, one can always instead reject some underlying "auxiliary hypothesis". The perfect example of this is how when experiments came out seeming to indicate neutrinos were moving faster than light no serious scientists actually believed the neutrinos moved faster than light, rather, all the scientists rightly believed that there must have been something wrong with the experiment.



Now to answer your questions.





  1. Are any of Newton's three laws considered to be 'falsified theories' by any 'working physicists'? If so, what evidence do they have that they believe falsifies those three theories?




Despite what I said above the answer to your question is yes. This is because 'working physicists' are generally not good philosophers of science and many 'working physicists' incorrectly think Poppers program of falsification is correct. Working physicists aren't good philosophers of science because philosophy of science doesn't really help them do their job better and they simply may not find it that interesting, so if they hold misconceptions about philosophy of science it doesn't cause any problem whatsoever in their daily work.





  1. If the three laws are still unfalsified, are there any other concepts that form a part of "Newtonian Mechanics" that we consider to be falsified?




No. As I said above no physical theory can be falsified.



Here's some information about Imre Lakatos who has a better philosophy of science in my opinion than Popper.






share|cite|improve this answer









$endgroup$









  • 3




    $begingroup$
    I carry no water for Popper, but a statement like "no scientific theory is ever falsified" does not seem to me to be an actual description of what scientists think and do (as you have noted), and I question a theory of science that says that most scientists don't do science.
    $endgroup$
    – Rococo
    Apr 13 at 19:40






  • 2




    $begingroup$
    @Rococo I won't make any claim as to what percentage of scientists will say falsification is how science works because I've never done or seen a survey. I will point out that one can be a productive scientists regardless of ones opinions about philosophy of science. I would argue that all good scientists (even those who believe in falsification) do not actually do their science by trying to falsify things. Instead they do what all good scientists do:
    $endgroup$
    – jgerber
    Apr 13 at 21:37






  • 1




    $begingroup$
    They mess around with their theories and experiments seeing which theories work under which conditions, they try to figure out why and when theories break down, they generate new theories if necessary. The question is: does this theory explain what I am seeing? Why? Why not? What does it mean if this other theory also describes what I am seeing?
    $endgroup$
    – jgerber
    Apr 13 at 21:40






  • 1




    $begingroup$
    No where did I say that most scientists don't do science. All I said was that many scientists have misconceptions about philosophy of science but I was careful to point out that a scientists thoughts on philosophy of science have very little bearing on his or her aptitude as a scientist.
    $endgroup$
    – jgerber
    Apr 13 at 21:42










  • $begingroup$
    I apologize for mischaracterizing your position. Nonetheless, I am not personally convinced that, for example, 'all good scientists (even those who believe in falsification) do not actually do their science by trying to falsify things.' To be clear, I would neither make the extreme opposing claim that science is all about falsifying theories.
    $endgroup$
    – Rococo
    Apr 13 at 22:56
















1












$begingroup$

First of all no scientific theory can possibly be falsified. Popper was wrong. See the Quine-Duhem thesis which says that instead of rejecting the theory when a seemingly falsifying experiment occurs, one can always instead reject some underlying "auxiliary hypothesis". The perfect example of this is how when experiments came out seeming to indicate neutrinos were moving faster than light no serious scientists actually believed the neutrinos moved faster than light, rather, all the scientists rightly believed that there must have been something wrong with the experiment.



Now to answer your questions.





  1. Are any of Newton's three laws considered to be 'falsified theories' by any 'working physicists'? If so, what evidence do they have that they believe falsifies those three theories?




Despite what I said above the answer to your question is yes. This is because 'working physicists' are generally not good philosophers of science and many 'working physicists' incorrectly think Poppers program of falsification is correct. Working physicists aren't good philosophers of science because philosophy of science doesn't really help them do their job better and they simply may not find it that interesting, so if they hold misconceptions about philosophy of science it doesn't cause any problem whatsoever in their daily work.





  1. If the three laws are still unfalsified, are there any other concepts that form a part of "Newtonian Mechanics" that we consider to be falsified?




No. As I said above no physical theory can be falsified.



Here's some information about Imre Lakatos who has a better philosophy of science in my opinion than Popper.






share|cite|improve this answer









$endgroup$









  • 3




    $begingroup$
    I carry no water for Popper, but a statement like "no scientific theory is ever falsified" does not seem to me to be an actual description of what scientists think and do (as you have noted), and I question a theory of science that says that most scientists don't do science.
    $endgroup$
    – Rococo
    Apr 13 at 19:40






  • 2




    $begingroup$
    @Rococo I won't make any claim as to what percentage of scientists will say falsification is how science works because I've never done or seen a survey. I will point out that one can be a productive scientists regardless of ones opinions about philosophy of science. I would argue that all good scientists (even those who believe in falsification) do not actually do their science by trying to falsify things. Instead they do what all good scientists do:
    $endgroup$
    – jgerber
    Apr 13 at 21:37






  • 1




    $begingroup$
    They mess around with their theories and experiments seeing which theories work under which conditions, they try to figure out why and when theories break down, they generate new theories if necessary. The question is: does this theory explain what I am seeing? Why? Why not? What does it mean if this other theory also describes what I am seeing?
    $endgroup$
    – jgerber
    Apr 13 at 21:40






  • 1




    $begingroup$
    No where did I say that most scientists don't do science. All I said was that many scientists have misconceptions about philosophy of science but I was careful to point out that a scientists thoughts on philosophy of science have very little bearing on his or her aptitude as a scientist.
    $endgroup$
    – jgerber
    Apr 13 at 21:42










  • $begingroup$
    I apologize for mischaracterizing your position. Nonetheless, I am not personally convinced that, for example, 'all good scientists (even those who believe in falsification) do not actually do their science by trying to falsify things.' To be clear, I would neither make the extreme opposing claim that science is all about falsifying theories.
    $endgroup$
    – Rococo
    Apr 13 at 22:56














1












1








1





$begingroup$

First of all no scientific theory can possibly be falsified. Popper was wrong. See the Quine-Duhem thesis which says that instead of rejecting the theory when a seemingly falsifying experiment occurs, one can always instead reject some underlying "auxiliary hypothesis". The perfect example of this is how when experiments came out seeming to indicate neutrinos were moving faster than light no serious scientists actually believed the neutrinos moved faster than light, rather, all the scientists rightly believed that there must have been something wrong with the experiment.



Now to answer your questions.





  1. Are any of Newton's three laws considered to be 'falsified theories' by any 'working physicists'? If so, what evidence do they have that they believe falsifies those three theories?




Despite what I said above the answer to your question is yes. This is because 'working physicists' are generally not good philosophers of science and many 'working physicists' incorrectly think Poppers program of falsification is correct. Working physicists aren't good philosophers of science because philosophy of science doesn't really help them do their job better and they simply may not find it that interesting, so if they hold misconceptions about philosophy of science it doesn't cause any problem whatsoever in their daily work.





  1. If the three laws are still unfalsified, are there any other concepts that form a part of "Newtonian Mechanics" that we consider to be falsified?




No. As I said above no physical theory can be falsified.



Here's some information about Imre Lakatos who has a better philosophy of science in my opinion than Popper.






share|cite|improve this answer









$endgroup$



First of all no scientific theory can possibly be falsified. Popper was wrong. See the Quine-Duhem thesis which says that instead of rejecting the theory when a seemingly falsifying experiment occurs, one can always instead reject some underlying "auxiliary hypothesis". The perfect example of this is how when experiments came out seeming to indicate neutrinos were moving faster than light no serious scientists actually believed the neutrinos moved faster than light, rather, all the scientists rightly believed that there must have been something wrong with the experiment.



Now to answer your questions.





  1. Are any of Newton's three laws considered to be 'falsified theories' by any 'working physicists'? If so, what evidence do they have that they believe falsifies those three theories?




Despite what I said above the answer to your question is yes. This is because 'working physicists' are generally not good philosophers of science and many 'working physicists' incorrectly think Poppers program of falsification is correct. Working physicists aren't good philosophers of science because philosophy of science doesn't really help them do their job better and they simply may not find it that interesting, so if they hold misconceptions about philosophy of science it doesn't cause any problem whatsoever in their daily work.





  1. If the three laws are still unfalsified, are there any other concepts that form a part of "Newtonian Mechanics" that we consider to be falsified?




No. As I said above no physical theory can be falsified.



Here's some information about Imre Lakatos who has a better philosophy of science in my opinion than Popper.







share|cite|improve this answer












share|cite|improve this answer



share|cite|improve this answer










answered Apr 13 at 18:54









jgerberjgerber

2,4021522




2,4021522








  • 3




    $begingroup$
    I carry no water for Popper, but a statement like "no scientific theory is ever falsified" does not seem to me to be an actual description of what scientists think and do (as you have noted), and I question a theory of science that says that most scientists don't do science.
    $endgroup$
    – Rococo
    Apr 13 at 19:40






  • 2




    $begingroup$
    @Rococo I won't make any claim as to what percentage of scientists will say falsification is how science works because I've never done or seen a survey. I will point out that one can be a productive scientists regardless of ones opinions about philosophy of science. I would argue that all good scientists (even those who believe in falsification) do not actually do their science by trying to falsify things. Instead they do what all good scientists do:
    $endgroup$
    – jgerber
    Apr 13 at 21:37






  • 1




    $begingroup$
    They mess around with their theories and experiments seeing which theories work under which conditions, they try to figure out why and when theories break down, they generate new theories if necessary. The question is: does this theory explain what I am seeing? Why? Why not? What does it mean if this other theory also describes what I am seeing?
    $endgroup$
    – jgerber
    Apr 13 at 21:40






  • 1




    $begingroup$
    No where did I say that most scientists don't do science. All I said was that many scientists have misconceptions about philosophy of science but I was careful to point out that a scientists thoughts on philosophy of science have very little bearing on his or her aptitude as a scientist.
    $endgroup$
    – jgerber
    Apr 13 at 21:42










  • $begingroup$
    I apologize for mischaracterizing your position. Nonetheless, I am not personally convinced that, for example, 'all good scientists (even those who believe in falsification) do not actually do their science by trying to falsify things.' To be clear, I would neither make the extreme opposing claim that science is all about falsifying theories.
    $endgroup$
    – Rococo
    Apr 13 at 22:56














  • 3




    $begingroup$
    I carry no water for Popper, but a statement like "no scientific theory is ever falsified" does not seem to me to be an actual description of what scientists think and do (as you have noted), and I question a theory of science that says that most scientists don't do science.
    $endgroup$
    – Rococo
    Apr 13 at 19:40






  • 2




    $begingroup$
    @Rococo I won't make any claim as to what percentage of scientists will say falsification is how science works because I've never done or seen a survey. I will point out that one can be a productive scientists regardless of ones opinions about philosophy of science. I would argue that all good scientists (even those who believe in falsification) do not actually do their science by trying to falsify things. Instead they do what all good scientists do:
    $endgroup$
    – jgerber
    Apr 13 at 21:37






  • 1




    $begingroup$
    They mess around with their theories and experiments seeing which theories work under which conditions, they try to figure out why and when theories break down, they generate new theories if necessary. The question is: does this theory explain what I am seeing? Why? Why not? What does it mean if this other theory also describes what I am seeing?
    $endgroup$
    – jgerber
    Apr 13 at 21:40






  • 1




    $begingroup$
    No where did I say that most scientists don't do science. All I said was that many scientists have misconceptions about philosophy of science but I was careful to point out that a scientists thoughts on philosophy of science have very little bearing on his or her aptitude as a scientist.
    $endgroup$
    – jgerber
    Apr 13 at 21:42










  • $begingroup$
    I apologize for mischaracterizing your position. Nonetheless, I am not personally convinced that, for example, 'all good scientists (even those who believe in falsification) do not actually do their science by trying to falsify things.' To be clear, I would neither make the extreme opposing claim that science is all about falsifying theories.
    $endgroup$
    – Rococo
    Apr 13 at 22:56








3




3




$begingroup$
I carry no water for Popper, but a statement like "no scientific theory is ever falsified" does not seem to me to be an actual description of what scientists think and do (as you have noted), and I question a theory of science that says that most scientists don't do science.
$endgroup$
– Rococo
Apr 13 at 19:40




$begingroup$
I carry no water for Popper, but a statement like "no scientific theory is ever falsified" does not seem to me to be an actual description of what scientists think and do (as you have noted), and I question a theory of science that says that most scientists don't do science.
$endgroup$
– Rococo
Apr 13 at 19:40




2




2




$begingroup$
@Rococo I won't make any claim as to what percentage of scientists will say falsification is how science works because I've never done or seen a survey. I will point out that one can be a productive scientists regardless of ones opinions about philosophy of science. I would argue that all good scientists (even those who believe in falsification) do not actually do their science by trying to falsify things. Instead they do what all good scientists do:
$endgroup$
– jgerber
Apr 13 at 21:37




$begingroup$
@Rococo I won't make any claim as to what percentage of scientists will say falsification is how science works because I've never done or seen a survey. I will point out that one can be a productive scientists regardless of ones opinions about philosophy of science. I would argue that all good scientists (even those who believe in falsification) do not actually do their science by trying to falsify things. Instead they do what all good scientists do:
$endgroup$
– jgerber
Apr 13 at 21:37




1




1




$begingroup$
They mess around with their theories and experiments seeing which theories work under which conditions, they try to figure out why and when theories break down, they generate new theories if necessary. The question is: does this theory explain what I am seeing? Why? Why not? What does it mean if this other theory also describes what I am seeing?
$endgroup$
– jgerber
Apr 13 at 21:40




$begingroup$
They mess around with their theories and experiments seeing which theories work under which conditions, they try to figure out why and when theories break down, they generate new theories if necessary. The question is: does this theory explain what I am seeing? Why? Why not? What does it mean if this other theory also describes what I am seeing?
$endgroup$
– jgerber
Apr 13 at 21:40




1




1




$begingroup$
No where did I say that most scientists don't do science. All I said was that many scientists have misconceptions about philosophy of science but I was careful to point out that a scientists thoughts on philosophy of science have very little bearing on his or her aptitude as a scientist.
$endgroup$
– jgerber
Apr 13 at 21:42




$begingroup$
No where did I say that most scientists don't do science. All I said was that many scientists have misconceptions about philosophy of science but I was careful to point out that a scientists thoughts on philosophy of science have very little bearing on his or her aptitude as a scientist.
$endgroup$
– jgerber
Apr 13 at 21:42












$begingroup$
I apologize for mischaracterizing your position. Nonetheless, I am not personally convinced that, for example, 'all good scientists (even those who believe in falsification) do not actually do their science by trying to falsify things.' To be clear, I would neither make the extreme opposing claim that science is all about falsifying theories.
$endgroup$
– Rococo
Apr 13 at 22:56




$begingroup$
I apologize for mischaracterizing your position. Nonetheless, I am not personally convinced that, for example, 'all good scientists (even those who believe in falsification) do not actually do their science by trying to falsify things.' To be clear, I would neither make the extreme opposing claim that science is all about falsifying theories.
$endgroup$
– Rococo
Apr 13 at 22:56











1












$begingroup$

No, they are not considered falsified. They are still a valid low energy approximation, which is all they ever were claimed to be. Or, if you must, they have been falsified at very high energies only.






share|cite|improve this answer









$endgroup$









  • 2




    $begingroup$
    Did Newton claim his laws were a low energy approximation? I doubt it.
    $endgroup$
    – immibis
    2 days ago










  • $begingroup$
    @immibis he claimed his laws explained the solar system. That claim has not been falsified apart from extremely tiny deviations.
    $endgroup$
    – my2cts
    2 days ago








  • 1




    $begingroup$
    @my2cts Newtonian mechanics wasn't created to work "apart from extremely tiny derivations". It was created and applied to work at all scales. That's why we had to develop beyond classical/Newtonian mechanics when the model was no longer accurate. They were not as widely applicable as was originally thought.
    $endgroup$
    – JMac
    2 days ago










  • $begingroup$
    @JMac "as originally thought" Can you tell where in "Opticks" speeds of the order of 300.000 km/s are considered? Where an accuracy of 43 arcsec per century was claimed?
    $endgroup$
    – my2cts
    yesterday










  • $begingroup$
    @my2cts Can you show where he mentions that it's only exact when velocity is zero? I don't believe he states anywhere the assumptions that this doesn't apply at particular scales, or anything of the sort. All the evidence suggests that the physical laws he derived were to apply at all scales and velocities, and none of what he wrote that I'm aware of ever tried to mention that it was only an approximation for low energy.
    $endgroup$
    – JMac
    yesterday
















1












$begingroup$

No, they are not considered falsified. They are still a valid low energy approximation, which is all they ever were claimed to be. Or, if you must, they have been falsified at very high energies only.






share|cite|improve this answer









$endgroup$









  • 2




    $begingroup$
    Did Newton claim his laws were a low energy approximation? I doubt it.
    $endgroup$
    – immibis
    2 days ago










  • $begingroup$
    @immibis he claimed his laws explained the solar system. That claim has not been falsified apart from extremely tiny deviations.
    $endgroup$
    – my2cts
    2 days ago








  • 1




    $begingroup$
    @my2cts Newtonian mechanics wasn't created to work "apart from extremely tiny derivations". It was created and applied to work at all scales. That's why we had to develop beyond classical/Newtonian mechanics when the model was no longer accurate. They were not as widely applicable as was originally thought.
    $endgroup$
    – JMac
    2 days ago










  • $begingroup$
    @JMac "as originally thought" Can you tell where in "Opticks" speeds of the order of 300.000 km/s are considered? Where an accuracy of 43 arcsec per century was claimed?
    $endgroup$
    – my2cts
    yesterday










  • $begingroup$
    @my2cts Can you show where he mentions that it's only exact when velocity is zero? I don't believe he states anywhere the assumptions that this doesn't apply at particular scales, or anything of the sort. All the evidence suggests that the physical laws he derived were to apply at all scales and velocities, and none of what he wrote that I'm aware of ever tried to mention that it was only an approximation for low energy.
    $endgroup$
    – JMac
    yesterday














1












1








1





$begingroup$

No, they are not considered falsified. They are still a valid low energy approximation, which is all they ever were claimed to be. Or, if you must, they have been falsified at very high energies only.






share|cite|improve this answer









$endgroup$



No, they are not considered falsified. They are still a valid low energy approximation, which is all they ever were claimed to be. Or, if you must, they have been falsified at very high energies only.







share|cite|improve this answer












share|cite|improve this answer



share|cite|improve this answer










answered Apr 14 at 22:55









my2ctsmy2cts

5,8042719




5,8042719








  • 2




    $begingroup$
    Did Newton claim his laws were a low energy approximation? I doubt it.
    $endgroup$
    – immibis
    2 days ago










  • $begingroup$
    @immibis he claimed his laws explained the solar system. That claim has not been falsified apart from extremely tiny deviations.
    $endgroup$
    – my2cts
    2 days ago








  • 1




    $begingroup$
    @my2cts Newtonian mechanics wasn't created to work "apart from extremely tiny derivations". It was created and applied to work at all scales. That's why we had to develop beyond classical/Newtonian mechanics when the model was no longer accurate. They were not as widely applicable as was originally thought.
    $endgroup$
    – JMac
    2 days ago










  • $begingroup$
    @JMac "as originally thought" Can you tell where in "Opticks" speeds of the order of 300.000 km/s are considered? Where an accuracy of 43 arcsec per century was claimed?
    $endgroup$
    – my2cts
    yesterday










  • $begingroup$
    @my2cts Can you show where he mentions that it's only exact when velocity is zero? I don't believe he states anywhere the assumptions that this doesn't apply at particular scales, or anything of the sort. All the evidence suggests that the physical laws he derived were to apply at all scales and velocities, and none of what he wrote that I'm aware of ever tried to mention that it was only an approximation for low energy.
    $endgroup$
    – JMac
    yesterday














  • 2




    $begingroup$
    Did Newton claim his laws were a low energy approximation? I doubt it.
    $endgroup$
    – immibis
    2 days ago










  • $begingroup$
    @immibis he claimed his laws explained the solar system. That claim has not been falsified apart from extremely tiny deviations.
    $endgroup$
    – my2cts
    2 days ago








  • 1




    $begingroup$
    @my2cts Newtonian mechanics wasn't created to work "apart from extremely tiny derivations". It was created and applied to work at all scales. That's why we had to develop beyond classical/Newtonian mechanics when the model was no longer accurate. They were not as widely applicable as was originally thought.
    $endgroup$
    – JMac
    2 days ago










  • $begingroup$
    @JMac "as originally thought" Can you tell where in "Opticks" speeds of the order of 300.000 km/s are considered? Where an accuracy of 43 arcsec per century was claimed?
    $endgroup$
    – my2cts
    yesterday










  • $begingroup$
    @my2cts Can you show where he mentions that it's only exact when velocity is zero? I don't believe he states anywhere the assumptions that this doesn't apply at particular scales, or anything of the sort. All the evidence suggests that the physical laws he derived were to apply at all scales and velocities, and none of what he wrote that I'm aware of ever tried to mention that it was only an approximation for low energy.
    $endgroup$
    – JMac
    yesterday








2




2




$begingroup$
Did Newton claim his laws were a low energy approximation? I doubt it.
$endgroup$
– immibis
2 days ago




$begingroup$
Did Newton claim his laws were a low energy approximation? I doubt it.
$endgroup$
– immibis
2 days ago












$begingroup$
@immibis he claimed his laws explained the solar system. That claim has not been falsified apart from extremely tiny deviations.
$endgroup$
– my2cts
2 days ago






$begingroup$
@immibis he claimed his laws explained the solar system. That claim has not been falsified apart from extremely tiny deviations.
$endgroup$
– my2cts
2 days ago






1




1




$begingroup$
@my2cts Newtonian mechanics wasn't created to work "apart from extremely tiny derivations". It was created and applied to work at all scales. That's why we had to develop beyond classical/Newtonian mechanics when the model was no longer accurate. They were not as widely applicable as was originally thought.
$endgroup$
– JMac
2 days ago




$begingroup$
@my2cts Newtonian mechanics wasn't created to work "apart from extremely tiny derivations". It was created and applied to work at all scales. That's why we had to develop beyond classical/Newtonian mechanics when the model was no longer accurate. They were not as widely applicable as was originally thought.
$endgroup$
– JMac
2 days ago












$begingroup$
@JMac "as originally thought" Can you tell where in "Opticks" speeds of the order of 300.000 km/s are considered? Where an accuracy of 43 arcsec per century was claimed?
$endgroup$
– my2cts
yesterday




$begingroup$
@JMac "as originally thought" Can you tell where in "Opticks" speeds of the order of 300.000 km/s are considered? Where an accuracy of 43 arcsec per century was claimed?
$endgroup$
– my2cts
yesterday












$begingroup$
@my2cts Can you show where he mentions that it's only exact when velocity is zero? I don't believe he states anywhere the assumptions that this doesn't apply at particular scales, or anything of the sort. All the evidence suggests that the physical laws he derived were to apply at all scales and velocities, and none of what he wrote that I'm aware of ever tried to mention that it was only an approximation for low energy.
$endgroup$
– JMac
yesterday




$begingroup$
@my2cts Can you show where he mentions that it's only exact when velocity is zero? I don't believe he states anywhere the assumptions that this doesn't apply at particular scales, or anything of the sort. All the evidence suggests that the physical laws he derived were to apply at all scales and velocities, and none of what he wrote that I'm aware of ever tried to mention that it was only an approximation for low energy.
$endgroup$
– JMac
yesterday










Pod is a new contributor. Be nice, and check out our Code of Conduct.










draft saved

draft discarded


















Pod is a new contributor. Be nice, and check out our Code of Conduct.













Pod is a new contributor. Be nice, and check out our Code of Conduct.












Pod is a new contributor. Be nice, and check out our Code of Conduct.
















Thanks for contributing an answer to Physics Stack Exchange!


  • Please be sure to answer the question. Provide details and share your research!

But avoid



  • Asking for help, clarification, or responding to other answers.

  • Making statements based on opinion; back them up with references or personal experience.


Use MathJax to format equations. MathJax reference.


To learn more, see our tips on writing great answers.




draft saved


draft discarded














StackExchange.ready(
function () {
StackExchange.openid.initPostLogin('.new-post-login', 'https%3a%2f%2fphysics.stackexchange.com%2fquestions%2f472215%2fdo-working-physicists-consider-newtonian-mechanics-to-be-falsified%23new-answer', 'question_page');
}
);

Post as a guest















Required, but never shown





















































Required, but never shown














Required, but never shown












Required, but never shown







Required, but never shown

































Required, but never shown














Required, but never shown












Required, but never shown







Required, but never shown







Popular posts from this blog

Plaza Victoria

In PowerPoint, is there a keyboard shortcut for bulleted / numbered list?

How to put 3 figures in Latex with 2 figures side by side and 1 below these side by side images but in...